Proving "Limits of Finite Sequences Implies Limit of Sum

In summary, the problem states that for a given sequence of finite numbers, if their sum is equal to 1, then the limit of the sequence multiplied by another convergent sequence is equal to the limit of the convergent sequence. The first direction is to show that if for every m in the sequence, the limit of the sequence is equal to 0, then the original statement holds true. The second direction is to prove the opposite, where if there exists an m where the limit is not equal to 0, then the original statement is false.
  • #1
Mr Davis 97
1,462
44

Homework Statement


For each ##n\in\mathbb{N}##, let the finite sequence ##\{b_{n,m}\}_{m=1}^n\subset(0,\infty)## be given. Assume, for each ##n\in\mathbb{N}##, that ##b_{n,1}+b_{n,2}+\cdots+b_{n,n}=1##.

Show that ##\lim_{n\to\infty}( b_{n,1}\cdot a_1+b_{n,2}\cdot a_2+\cdots+b_{n,n}\cdot a_n) = \lim_{n\to\infty}a_n##, for every convergent sequence ##\{a_n\}_{n=1}^\infty\subset\mathbb{R}## if and only if, for each ##m\in\mathbb{N}##, ##\lim_{n\to\infty}b_{n,m}=0##.

Homework Equations

The Attempt at a Solution


I really need at least one hint for this one. Which direction of the proof should I start with? Which one is easier?

My idea for the <---- direction is that since for each ##m\in\mathbb{N}##, ##\lim_{n\to\infty}b_{n,m}=0##, we see that ## \lim_{n\to\infty}b_{n,1}\cdot a_1+\lim_{n\to\infty}b_{n,2}\cdot a_2+\cdots+\lim_{n\to\infty}b_{n,n}\cdot a_n) = 0 + 0 + \cdots + 0##... But I'm not sure where this gets me.
 
Physics news on Phys.org
  • #2
Mr Davis 97 said:

Homework Statement


For each ##n\in\mathbb{N}##, let the finite sequence ##\{b_{n,m}\}_{m=1}^n\subset(0,\infty)## be given. Assume, for each ##n\in\mathbb{N}##, that ##b_{n,1}+b_{n,2}+\cdots+b_{n,n}=1##.

Show that ##\lim_{n\to\infty}( b_{n,1}\cdot a_1+b_{n,2}\cdot a_2+\cdots+b_{n,n}\cdot a_n) = \lim_{n\to\infty}a_n##, for every convergent sequence ##\{a_n\}_{n=1}^\infty\subset\mathbb{R}## if and only if, for each ##m\in\mathbb{N}##, ##\lim_{n\to\infty}b_{n,m}=0##.

Homework Equations

The Attempt at a Solution


I really need at least one hint for this one. Which direction of the proof should I start with? Which one is easier?

My idea for the <---- direction is that since for each ##m\in\mathbb{N}##, ##\lim_{n\to\infty}b_{n,m}=0##, we see that ## \lim_{n\to\infty}b_{n,1}\cdot a_1+\lim_{n\to\infty}b_{n,2}\cdot a_2+\cdots+\lim_{n\to\infty}b_{n,n}\cdot a_n) = 0 + 0 + \cdots + 0##... But I'm not sure where this gets me.
Here is my stab at a solution for the <--- direction.

Let ##(a_n)## be an arbitrary convergent sequence that converges to ##c##. We want to show that ##b_{n,1}a_n + \cdots b_{n,n}a_n## also converges to ##c##. To this end, given ##\epsilon > 0##, there exists ##N \in \mathbb{N}## such that if ##n \ge N## we have ##|a_n - c| \le \epsilon##. ##|b_{n,1} a_1 + \cdots b_{n,n}a_n - c| = |b_{n,1} a_1 + \cdots + b_{n,n}a_n - c(b_{n,1}+ \cdots + b_{n,n})| = |b_{n,1}(a_1 - c) + \cdots b_{n,n}(a_n - c)|##... This is kind of where I get stuck. I'm not sure what to do next.
 
  • #3
Mr Davis 97 said:
My idea for the <---- direction is that since for each ##m\in\mathbb{N}##, ##\lim_{n\to\infty}b_{n,m}=0##, we see that ## \lim_{n\to\infty}b_{n,1}\cdot a_1+\lim_{n\to\infty}b_{n,2}\cdot a_2+\cdots+\lim_{n\to\infty}b_{n,n}\cdot a_n) = 0 + 0 + \cdots + 0##... But I'm not sure where this gets me.
You cannot split a limit like that.

The other direction is easier if you make a proof by contradiction. Assume there is an m such that ##\lim_{n\to\infty}b_{n,m} \neq 0## and show that the original statement is no longer true. That should also give an idea how the overall structure works.
 

1. What is the definition of a "limit of a finite sequence"?

A limit of a finite sequence is the value that the sequence approaches as its index approaches infinity. In other words, it is the value towards which the terms of the sequence get closer and closer, without ever reaching it.

2. How is the limit of a finite sequence related to the limit of the sum of its terms?

The limit of a finite sequence is directly related to the limit of the sum of its terms. If the limit of the sum of the terms exists, then the limit of the sequence also exists and is equal to the same value. This means that the sum of the terms can be used to determine the limit of a finite sequence.

3. Why do we need to prove the "Limits of Finite Sequences Implies Limit of Sum" theorem?

This theorem is important because it allows us to simplify the process of finding the limit of a finite sequence. Instead of evaluating each term individually, we can just find the limit of the sum of the terms, which is often easier to do. This theorem also helps us better understand the relationship between limits and sums.

4. What are some examples of finite sequences where this theorem can be applied?

This theorem can be applied to any finite sequence where the limit of the sum of its terms exists. Some examples include arithmetic sequences, geometric sequences, and harmonic sequences. It can also be applied to more complex sequences, such as alternating sequences or recursively defined sequences.

5. Are there any exceptions to the "Limits of Finite Sequences Implies Limit of Sum" theorem?

Yes, there are a few exceptions to this theorem. One exception is when the terms of the sequence are not defined for all indices, in which case the limit of the sum does not exist. Another exception is when the terms of the sequence are not all positive, in which case the limit of the sum may not exist. Lastly, this theorem may not apply to infinite sequences, as the limit of the sum may not be defined in that case.

Similar threads

  • Calculus and Beyond Homework Help
Replies
7
Views
1K
  • Calculus and Beyond Homework Help
Replies
8
Views
820
  • Calculus and Beyond Homework Help
Replies
8
Views
667
  • Calculus and Beyond Homework Help
Replies
2
Views
712
  • Calculus and Beyond Homework Help
Replies
5
Views
489
  • Calculus and Beyond Homework Help
Replies
1
Views
261
  • Calculus and Beyond Homework Help
Replies
34
Views
2K
  • Calculus and Beyond Homework Help
Replies
2
Views
190
  • Calculus and Beyond Homework Help
Replies
7
Views
1K
  • Calculus and Beyond Homework Help
Replies
13
Views
691
Back
Top